HTN Flashcards

1
Q
  1. A 55-year-old white man with seated office blood pressure (BP) readings of 144/92mm Hg and 136/84 mm Hg is asked to return in 2 weeks for repeat measurements, which are 138/88 mm Hg and 134/82 mm Hg. Which of the following classifies DG’s BP per the American Society of Hypertension (ASH) and the International Society of Hypertension (ISH) joint Clinical Practice Guidelines for the Management of Hypertension in the Community?

A. Isolated systolic hypertension
B. Stage 1 hypertension
C. Prehypertension
D. Optimal BP
E. Stage 2 hypertension

A

C

How well did you know this?
1
Not at all
2
3
4
5
Perfectly
2
Q
  1. Lupus-like syndrome is a possible side effect of which of the following drug(s)?

A. Clonidine
B. Minoxidil
C. Doxazosin
D. Hydralazine
E. Reserpine

A

D

How well did you know this?
1
Not at all
2
3
4
5
Perfectly
3
Q
  1. A 55-year-old black woman has a history of left ventricular hypertrophy with a left
    ventricular ejection fraction of 55%. She has had hypertension for 10 years and is
    currently taking chlorthalidone 25 mg daily, metoprolol succinate 50 mg daily, and
    amlodipine 2.5 mg daily. Her averaged BP is 152/94 mm Hg with a heart rate of 54
    beats/min. Her physical exam is unremarkable and basic metabolic panel reveals serum creatinine of 0.8 mg/dL [71 μmol/L] and potassium of 3.9 mEq/L (3.9
    mmol/L). She reports allergies to fosinopril and aspirin. Which of the following
    represents the optimal course of action?

A. Increase amlodipine to 5 mg and have her take it at bedtime
B. Increase metoprolol succinate to 100 mg daily
C. Add lisinopril 5 mg daily
D. Add spironolactone 50 mg daily
E. A or B

A

A

How well did you know this?
1
Not at all
2
3
4
5
Perfectly
4
Q
  1. A 34-year-old black man presents to your clinic with a BP of 160/94 mm Hg.
    Repeat readings over the next 2 weeks average 156/92 mm Hg. The patient has no
    past medical history with the exception of Crohn disease, which is currently treated
    with chronic steroid therapy. He is also taking an over-the-counter NSAID for
    ongoing back pain. Physical examination and laboratory tests are unremarkable.
    Appropriate interventions at this time include:

A. No intervention because patient most likely has drug-induced hypertension
B. Discontinuation of the NSAID
C. Reassessment of the dose and need for long-ter

A

E

How well did you know this?
1
Not at all
2
3
4
5
Perfectly
5
Q
  1. A 68-year-old white man has resistant hypertension, prior myocardial infarction,
    and chronic kidney disease (CKD; serum creatinine 1.8 mg/dL [159 μmol/L],
    estimated creatinine clearance 40 mL/min [0.67 mL/s]). You are initiating ramipril
    today. What is the most appropriate timeframe for laboratory follow-up?

A. 1 to 2 days
B. 1 to 2 weeks
C. 1 to 2 months
D. 3 to 4 months
E. 4 to 6 months

A

B

How well did you know this?
1
Not at all
2
3
4
5
Perfectly
6
Q
  1. A 47-year-old Hispanic man has primary hypertension with an average BP of
    172/98 mm Hg and heart rate of 70 beats/min. His most recent serum potassium is 4.5 mEq/L (4.5 mmol/L), serum creatinine is 1.1 mg/dL (97 μmol/L) and calculated creatinine clearance is 102 mL/min (1.70 mL/s). Which of the following
    antihypertensives would be most appropriate at this time?

A. Furosemide
B. Atenolol
C. Chlorthalidone and lisinopril initiated concurrently
D. Amlodipine and lisinopril initiated concurrently
E. C or D

A

E

How well did you know this?
1
Not at all
2
3
4
5
Perfectly
7
Q
  1. A 67-year-old Asian man with a recent non-ST segment elevation MI (2 weeks
    ago) has an average BP of 148/86 mm Hg and a heart rate of 76 beats/min. Which of
    the following antihypertensive agents is preferred in this setting?

A. Metoprolol tartrate
B. Acebutolol
C. Hydrochlorothiazide
D. Spironolactone
E. A or B

A

A

How well did you know this?
1
Not at all
2
3
4
5
Perfectly
8
Q
  1. Which of the following treatments is (are) the most appropriate for a hypertensive emergency?

A. Normalization of BP within hours
B. Reduction in mean arterial pressure by 25% to 50% within minutes to hours
C. Reduction in mean arterial pressure up to 25% within minutes to hours
D. Administration of sublingual nifedipine
E. C and D

A

C

How well did you know this?
1
Not at all
2
3
4
5
Perfectly
9
Q
  1. A 65-year-old black man with history of hypertension, prior MI, and benign
    prostatic hypertrophy, is currently receiving amlodipine 5 mg QAM and metoprolol succinate 50 mg once daily. He has an average 24-hour Ambulatory Blood Pressure of 156/92 mm Hg and HR of 66 beats/min with notable nocturnal hypertension. He complains of nocturia but states that the swelling in his feet improved when his amlodipine dose was reduced. Which of the following presents the most clinically appropriate course of action?

A. Initiate tamsulosin 0.4 mg daily at bedtime
B. Increase amlodipine to 10 mg daily and change to bedtime
C. Increase metoprolol succinate to 50 mg twice daily
D. Initiate chlorthalidone 50 mg daily at bedtime
E. Initiate doxazosin 2 mg daily at bedtime

A

E

How well did you know this?
1
Not at all
2
3
4
5
Perfectly
10
Q
  1. A 67-year-old black man has resistant hypertension. Past medical history is also
    significant for heart failure with left ventricular systolic dysfunction, dyslipidemia, and peripheral vascular disease. Medications currently include lisinopril, carvedilol, and furosemide. Current blood pressure is 146/88 mm Hg and when repeated 148/82mm Hg. Which of the following additions to his medication regimen would be an inappropriate choice at this time?

A. Amlodipine
B. Felodipine
C. Hydralazine/Isosorbide Dinitrate
D. Minoxidil
E. Spironolactone

A

D

How well did you know this?
1
Not at all
2
3
4
5
Perfectly
11
Q
  1. A 32-year-old woman is 20 weeks pregnant and has a history of gestational
    diabetes. She presents with an average BP of 154/96 mm Hg and a heart rate of 60
    beats/min. Her laboratory results are remarkable for proteinuria, elevated serum uric acid, and low potassium. Which of the following presents the most appropriate course of action?

A. Closely monitor her BP and provide supportive care
B. Start Losartan 50 mg daily while monitoring BP
C. Start methyldopa 250 mg every 6 hours while monitoring BP
D. Start labetalol 100 mg every 12 hours while monitoring BP
E. Start chlorthalidone 25 mg daily while monitoring BP

A

C

How well did you know this?
1
Not at all
2
3
4
5
Perfectly
12
Q
  1. A 45-year-old black man has a past medical history significant only for
    hypertension. Despite therapy with lisinopril 40 mg daily, hydrochlorothiazide 12.5mg daily, and amlodipine 10 mg daily, his home and office BPs over the last 2 weeks remain elevated with an average reading of 154/92 mm Hg. All laboratory results are within normal limits. Which of the following would be a reasonable change to his antihypertensive regimen?

A. Replace hydrochlorothiazide with chlorthalidone 25 mg daily
B. Add aliskiren 150 mg daily
C. Add spironolactone 50 mg daily
D. Add losartan 25 mg daily
E. B or D

A

A

How well did you know this?
1
Not at all
2
3
4
5
Perfectly
13
Q
  1. A 29-year-old woman has had stage 1 hypertension for the past 2 years that has
    been well controlled (BP range of 100–110/60–65 mm Hg) on lisinopril 10 mg once
    daily. She has successfully implemented lifestyle modifications, losing 14 kg (31 lb)
    and obtaining a body mass index of 21 kg/m2. She informs you she is going to start trying to get pregnant. What changes should be instituted with her antihypertensive therapy at this time?

A. Discontinuing lisinopril and monitoring BP closely with lifestyle modifications
B. Discontinuing lisinopril and initiating methyldopa
C. Continuing lisinopril because her BP is well controlled
D. Reducing lisinopril dose to 2.5 mg daily and maintaining lifestyle modifications
E. Discontinuing lisinopril and starting HCTZ

A

A

How well did you know this?
1
Not at all
2
3
4
5
Perfectly
14
Q
  1. A 57-year-old white woman has type 2 diabetes, morbid obesity, and hypertension. She is currently taking only lisinopril 20 mg daily and her office blood
    pressures are consistently at goal < 140/90 mm Hg, but her home readings are
    significantly higher. Which of the following is a possible explanation for her elevated
    home readings?

A. Her home BP cuff is too small
B. She has white coat hypertension
C. Her home BP cuff is too large
D. She checks her blood pressure immediately after exercise
E. All of the above

A

A

How well did you know this?
1
Not at all
2
3
4
5
Perfectly
15
Q
  1. A 56-year-old black woman is currently on verapamil ER 360 mg once daily. She
    has a past medical history of hypertension and atrial fibrillation. Today, her office BP
    readings are 137/97 mm Hg and 144/96 mm Hg with a heart rate of 60 beats/min.
    Which of the following is the most appropriate intervention?

A. Add amlodipine 5 mg daily
B. Increase verapamil ER to 360 mg twice daily
C. Add chlorthalidone 12.5 mg daily
D. Add lisinopril 5 mg daily
E. Either C or D

A

E

How well did you know this?
1
Not at all
2
3
4
5
Perfectly
16
Q
  1. A 55-year-old man with seated office blood pressure (BP) readings of 138/89 mm Hg and 136/84 mm Hg is asked to return in 2 weeks for repeat measurements, which are 138/88 mm Hg and 134/82 mm Hg. Which of the following classifies his BP per the 2017 ACC/AHA guidelines?

A. Stage 2 hypertension
B. Stage 1 hypertension
C. Elevated BP
D. Optimal BP

17
Q
  1. A 45-year-old woman who has resistant hypertension, heart failure with reduced ejection fraction, type 2 diabetes, and dyslipidemia comes to the clinic for a medication management visit. Her current medications include enalapril/hydrochlorothiazide (Vaseretic), metoprolol succinate, hydralazine, isosorbide dinitrate, metformin, and atorvastatin. She has been adherent
    to all her medications. She complains of recently developed fatigue, arthralgia, and mild peripheral edema, and a butterfly rash across her nose and upper cheeks. Which of the following statements is correct?

A. The most likely medication causing this adverse reaction is the angiotensin-converting enzyme inhibitor, enalapril.
B. This is likely to be a dose-independent adverse drug reaction.
C. Testing for N-acetyltransferase-1 (NAT1) and NAT2 genes may help to prevent this adverse effect.
D. Discontinue all medications due to unknown etiology of this reaction.

18
Q
  1. A 55-year-old woman has a history of left ventricular hypertrophy with a left ventricular ejection fraction of 55% (0.55). She has had hypertension for 10 years and is currently taking chlorthalidone 25 mg daily, metoprolol succinate 50 mg daily, and amlodipine 2.5 mg daily. Her
    averaged BP is 152/94 mm Hg with a heart rate of 54 beats/min. Her physical examination is unremarkable, and basic metabolic panel reveals serum creatinine of 0.8 mg/dL (71 µmol/L) and potassium of 3.9 mEq/L (mmol/L). She reports allergies to fosinopril and aspirin. Which of the
    following represents the optimal course of action?

A. Increase amlodipine to 5 mg and have her take it at bedtime
B. Increase metoprolol succinate to 100 mg daily
C. Add lisinopril 5 mg daily
D. Add spironolactone 50 mg daily

19
Q
  1. A 58-year-old man presents to urgent care with chief complaints of severe headache, confusion, and blurred vision. Upon examination, his vital signs and relevant laboratory values
    include BP 218/124 mm Hg; HR 92 beats/min; RR 18 breaths/min; serum potassium 4.8 mEq/L (mmol/L); serum creatinine 2.4 mg/dL (212 µmol/L); blood urea nitrogen 45 mg/dL (16.1
    mmol/L); serum glucose 145 mg/dL (8.0 mmol/L); hemoglobin A 1C 6.4% (0.064; 46 mmol/mol Hb); AST 28 U/L (0.47 µkat/L); ALT 20 U/L (0.33 µkat/L); urinalysis shows positive of proteins and negative of ketones. He has medical history of hypertension but no history of renal
    or hepatic impairment. According to the pharmacist, his antihypertensive medications are overdue for a refill by 2 months. Which of the following statements is most accurate?

A. He is experiencing hypertensive urgency.
B. Goal is to reduce SBP by up to 25% within an hour.
C. Goal is to reduce SBP by 25% to 50% within an hour.
D. Administration of short-acting oral antihypertensive such as labetalol or clonidine is appropriate.

20
Q
  1. A 57-year-old woman has type 2 diabetes, morbid obesity, and hypertension. She is currently
    taking only lisinopril 20 mg daily, and her office BPs are consistently at goal less than 130/80mm Hg, but her home readings are significantly higher. Which of the following is a possible explanation for her elevated home readings?

A. Her home BP cuff is too small.
B. She has white coat hypertension.
C. Her home BP cuff is too large.
D. She checks her BP immediately after exercise.

21
Q
  1. A 72-year-old black man with history of heart failure (NYHA class III symptoms) with reduced left ventricular ejection fraction, dyslipidemia, and peripheral arterial disease presents for risk factor reduction follow-up. His weight does not change from the baseline and he lacks
    any signs and symptoms for fluid retention. His BP in clinic is 142/75 mm Hg initially and 140/77 mm Hg on repeat. Current medications include bisoprolol 10 mg once daily and valsartan 160 mg twice daily along with furosemide 20 mg as needed for edema. Current laboratory values are stable and within normal limits with serum creatinine of 1.4 mg/dL (124 µmol/L) and potassium of 5.1 mEq/L (mmol/L). Which of the following additions to his medication regimen would be most appropriate at this time?

A. Amlodipine
B. Furosemide on a regular daily basis
C. Hydralazine/isosorbide dinitrate
D. Minoxidil

22
Q
  1. A 32-year-old woman is 20 weeks pregnant and has a history of gestational diabetes. She presents with an average BP of 154/96 mm Hg and a heart rate of 60 beats/min. Her laboratory results are remarkable for proteinuria, elevated serum uric acid, and low potassium. Which of the
    following presents the most appropriate course of action?

A. Closely monitor her BP and provide supportive care
B. Start losartan 50 mg daily while monitoring BP
C. Start methyldopa 250 mg every 6 hours while monitoring BP
D. Start labetalol 100 mg every 12 hours while monitoring BP

23
Q
  1. An 80-year-old man presents to your clinic today with average BP of 142/84 mm Hg. The patient is a community-dwelling adult with no significant past medical history and is in agreement to starting an antihypertensive agent today. Which of the following is the preferred guideline-based BP goal for this patient?

A. < 130/80 mm Hg
B. < 135/80 mm Hg
C. < 140/90 mm Hg
D. < 150/90 mm Hg

24
Q
  1. A 79-year-old man with a past medical history significant for hypertension and dyslipidemia returns for a 3-week follow-up on BP readings and laboratory values after initiation of lisinopril 10 mg daily for uncontrolled hypertension. Today, average BP has improved from 151/93 mm
    Hg to 128/71 mm Hg with an increase in serum creatinine from 1.2 mg/dL (106 μmol/L) to 1.4 mg/dL (124 μmol/L). Which of the following interventions would be recommended at this time?

A. Continue lisinopril 10 mg once daily
B. Stop lisinopril and initiate amlodipine 5 mg daily
C. Decrease lisinopril to 5 mg daily
D. Stop lisinopril and initiate losartan 25 mg daily

25
Q
  1. A 57-year-old man with a past medical history of hypertension, type 2 diabetes, and dyslipidemia returns to clinic for follow-up on BP readings after recent initiation of amlodipine.
    His current medications include amlodipine 5 mg daily, metformin, and atorvastatin. Per his office BP measurements and reported HBPM, there has been minimal change in his BP and it remains uncontrolled. Patient also reports new-onset edema of the lower extremities. Which of the following would be the most appropriate intervention at this time?

A. Continue amlodipine and initiate chlorthalidone 12.5 mg daily
B. Continue amlodipine and initiate furosemide 20 mg daily
C. Stop amlodipine and initiate lisinopril 10 mg daily
D. Stop amlodipine and initiate atenolol 25 mg daily

26
Q
  1. A 55-year-old woman with a past medical history significant for hypertension and 1+ bilateral edema is admitted due to decompensated heart failure with reduced ejection fraction of 30% to 35% (0.3–0.35). Her current medications include carvedilol 25 mg twice daily, furosemide 20 mg once daily, and amlodipine 5 mg daily. Her BP is currently controlled on this regimen with an average BP of 118/77 mm Hg and heart rate of 68 beats/min. She denies any
    allergies. What intervention would be recommended at this time?

A. No interventions necessary
B. Initiate lisinopril 10 mg daily
C. Discontinue amlodipine and initiate enalapril 5 mg once daily
D. Discontinue amlodipine and initiate chlorthalidone 12.5 mg daily

27
Q
  1. A 67-year-old woman with a past medical history significant for hypertension, dyslipidemia, and well-controlled asthma presents to clinic for postdischarge follow-up status post-MI. Her vital signs include an average BP of 139/87 mm Hg and HR of 85 beats/min. Her current medication list includes lisinopril, rosuvastatin, aspirin, clopidogrel, and budesonide-formoterol. Which of the following β-blockers would be recommended to initiate today?

A. Propranolol
B. Labetalol
C. Pindolol
D. Bisoprolol

28
Q
  1. What daytime ABPM target would be most reasonable to apply when assessing BP control with ABPM?

A. < 120/70 mm Hg
B. < 125/75 mm Hg
C. < 130/80 mm Hg
D. < 135/85 mm Hg

29
Q
  1. Initiation of hydrochlorothiazide can result in which of the following electrolyte disturbance?

A. Hyperkalemia
B. Hypouricemia
C. Hypercalcemia
D. Hypernatremia

30
Q
  1. A 56-year-old woman presents to your clinic today with an average BP of 137/88 mm Hg. She has no significant past medical history and is currently taking a multivitamin daily. Significant vital signs and laboratory values include HR 79 beats/min, BMI 30.1 kg/m 2 ,
    potassium 4.1 mEq/L (mmol/L), and an estimated GFR of 78 mL/min/1.73 m 2 . She has an estimated 10-year ASCVD risk of 7.5%. Which of the following would be the most appropriate intervention at this time?

A. Recommend weight loss, exercise, and a heart-healthy diet
B. Initiate lisinopril 10 mg daily
C. Initiate amlodipine 5 mg daily
D. Initiate labetalol 100 mg twice daily